Properties & Defintions of the First Uncountable Ordinal












0












$begingroup$


I am confused about the concept of the “first uncountable ordinal.” Here are 3 statements that are apparently equivalent:



(1) There is an uncountable ordinal that is “first” in the sense that it is the smallest ordinal that, when considered as a set, is uncountable.



(2) This ordinal is the supremum of all countable ordinals.



(3) The elements of the set are the countable ordinals, of which there are uncountable many.



First, which statement is generally given as the definition of “first uncountable ordinal”?



Secondly, how do we know the supremum in statement (2) exists?



Third, why does statement (3) follow from the other statements? And vice versa?










share|cite|improve this question











$endgroup$








  • 1




    $begingroup$
    en.wikipedia.org/wiki/Hartogs_number $leftarrow$ Use this with $X=omega$.
    $endgroup$
    – J.G.
    Jan 24 at 22:10












  • $begingroup$
    (3) Is just a consequence of the particular way we've defined ordinals and (2). Every ordinal is the set of all ordinals below it, and being a supremum of countable ordinals means the ones below it are exactly the countable ones. I leave this as a comment because I think it's the least substantial of the three.
    $endgroup$
    – Malice Vidrine
    Jan 24 at 23:45
















0












$begingroup$


I am confused about the concept of the “first uncountable ordinal.” Here are 3 statements that are apparently equivalent:



(1) There is an uncountable ordinal that is “first” in the sense that it is the smallest ordinal that, when considered as a set, is uncountable.



(2) This ordinal is the supremum of all countable ordinals.



(3) The elements of the set are the countable ordinals, of which there are uncountable many.



First, which statement is generally given as the definition of “first uncountable ordinal”?



Secondly, how do we know the supremum in statement (2) exists?



Third, why does statement (3) follow from the other statements? And vice versa?










share|cite|improve this question











$endgroup$








  • 1




    $begingroup$
    en.wikipedia.org/wiki/Hartogs_number $leftarrow$ Use this with $X=omega$.
    $endgroup$
    – J.G.
    Jan 24 at 22:10












  • $begingroup$
    (3) Is just a consequence of the particular way we've defined ordinals and (2). Every ordinal is the set of all ordinals below it, and being a supremum of countable ordinals means the ones below it are exactly the countable ones. I leave this as a comment because I think it's the least substantial of the three.
    $endgroup$
    – Malice Vidrine
    Jan 24 at 23:45














0












0








0





$begingroup$


I am confused about the concept of the “first uncountable ordinal.” Here are 3 statements that are apparently equivalent:



(1) There is an uncountable ordinal that is “first” in the sense that it is the smallest ordinal that, when considered as a set, is uncountable.



(2) This ordinal is the supremum of all countable ordinals.



(3) The elements of the set are the countable ordinals, of which there are uncountable many.



First, which statement is generally given as the definition of “first uncountable ordinal”?



Secondly, how do we know the supremum in statement (2) exists?



Third, why does statement (3) follow from the other statements? And vice versa?










share|cite|improve this question











$endgroup$




I am confused about the concept of the “first uncountable ordinal.” Here are 3 statements that are apparently equivalent:



(1) There is an uncountable ordinal that is “first” in the sense that it is the smallest ordinal that, when considered as a set, is uncountable.



(2) This ordinal is the supremum of all countable ordinals.



(3) The elements of the set are the countable ordinals, of which there are uncountable many.



First, which statement is generally given as the definition of “first uncountable ordinal”?



Secondly, how do we know the supremum in statement (2) exists?



Third, why does statement (3) follow from the other statements? And vice versa?







set-theory






share|cite|improve this question















share|cite|improve this question













share|cite|improve this question




share|cite|improve this question








edited Jan 25 at 1:19









Andrés E. Caicedo

65.6k8160250




65.6k8160250










asked Jan 24 at 21:47









MPittsMPitts

739412




739412








  • 1




    $begingroup$
    en.wikipedia.org/wiki/Hartogs_number $leftarrow$ Use this with $X=omega$.
    $endgroup$
    – J.G.
    Jan 24 at 22:10












  • $begingroup$
    (3) Is just a consequence of the particular way we've defined ordinals and (2). Every ordinal is the set of all ordinals below it, and being a supremum of countable ordinals means the ones below it are exactly the countable ones. I leave this as a comment because I think it's the least substantial of the three.
    $endgroup$
    – Malice Vidrine
    Jan 24 at 23:45














  • 1




    $begingroup$
    en.wikipedia.org/wiki/Hartogs_number $leftarrow$ Use this with $X=omega$.
    $endgroup$
    – J.G.
    Jan 24 at 22:10












  • $begingroup$
    (3) Is just a consequence of the particular way we've defined ordinals and (2). Every ordinal is the set of all ordinals below it, and being a supremum of countable ordinals means the ones below it are exactly the countable ones. I leave this as a comment because I think it's the least substantial of the three.
    $endgroup$
    – Malice Vidrine
    Jan 24 at 23:45








1




1




$begingroup$
en.wikipedia.org/wiki/Hartogs_number $leftarrow$ Use this with $X=omega$.
$endgroup$
– J.G.
Jan 24 at 22:10






$begingroup$
en.wikipedia.org/wiki/Hartogs_number $leftarrow$ Use this with $X=omega$.
$endgroup$
– J.G.
Jan 24 at 22:10














$begingroup$
(3) Is just a consequence of the particular way we've defined ordinals and (2). Every ordinal is the set of all ordinals below it, and being a supremum of countable ordinals means the ones below it are exactly the countable ones. I leave this as a comment because I think it's the least substantial of the three.
$endgroup$
– Malice Vidrine
Jan 24 at 23:45




$begingroup$
(3) Is just a consequence of the particular way we've defined ordinals and (2). Every ordinal is the set of all ordinals below it, and being a supremum of countable ordinals means the ones below it are exactly the countable ones. I leave this as a comment because I think it's the least substantial of the three.
$endgroup$
– Malice Vidrine
Jan 24 at 23:45










1 Answer
1






active

oldest

votes


















0












$begingroup$

(1) is the definition.



Since the ordinals are well ordered, one just needs to show there is an uncountable ordinal.

Use Hartogs lemma. Alternatively, use the axiom of choice to well order the reals.



Let u be the first uncountable ordinal.

If k is a countable ordinal, k < u.

So u is an upper bound of countable ordinals.

If d is an upper bound of the countable ordinals and d < u, then d is countable. As d + 1 is countable, d + 1 <= d, a contradiction.

So u is the supremum of the countable ordinals.






share|cite|improve this answer









$endgroup$













  • $begingroup$
    We don't need AC . Let $B$ be the set of subsets of $omega times omega$ that are (graphs of) well-orderings. Each $bin B$ is isomorphic to a $unique$ (countable) ordinal $F(b).$ Then $ {F(b): bin B} =omega_1$.
    $endgroup$
    – DanielWainfleet
    Jan 26 at 16:58











Your Answer





StackExchange.ifUsing("editor", function () {
return StackExchange.using("mathjaxEditing", function () {
StackExchange.MarkdownEditor.creationCallbacks.add(function (editor, postfix) {
StackExchange.mathjaxEditing.prepareWmdForMathJax(editor, postfix, [["$", "$"], ["\\(","\\)"]]);
});
});
}, "mathjax-editing");

StackExchange.ready(function() {
var channelOptions = {
tags: "".split(" "),
id: "69"
};
initTagRenderer("".split(" "), "".split(" "), channelOptions);

StackExchange.using("externalEditor", function() {
// Have to fire editor after snippets, if snippets enabled
if (StackExchange.settings.snippets.snippetsEnabled) {
StackExchange.using("snippets", function() {
createEditor();
});
}
else {
createEditor();
}
});

function createEditor() {
StackExchange.prepareEditor({
heartbeatType: 'answer',
autoActivateHeartbeat: false,
convertImagesToLinks: true,
noModals: true,
showLowRepImageUploadWarning: true,
reputationToPostImages: 10,
bindNavPrevention: true,
postfix: "",
imageUploader: {
brandingHtml: "Powered by u003ca class="icon-imgur-white" href="https://imgur.com/"u003eu003c/au003e",
contentPolicyHtml: "User contributions licensed under u003ca href="https://creativecommons.org/licenses/by-sa/3.0/"u003ecc by-sa 3.0 with attribution requiredu003c/au003e u003ca href="https://stackoverflow.com/legal/content-policy"u003e(content policy)u003c/au003e",
allowUrls: true
},
noCode: true, onDemand: true,
discardSelector: ".discard-answer"
,immediatelyShowMarkdownHelp:true
});


}
});














draft saved

draft discarded


















StackExchange.ready(
function () {
StackExchange.openid.initPostLogin('.new-post-login', 'https%3a%2f%2fmath.stackexchange.com%2fquestions%2f3086397%2fproperties-defintions-of-the-first-uncountable-ordinal%23new-answer', 'question_page');
}
);

Post as a guest















Required, but never shown

























1 Answer
1






active

oldest

votes








1 Answer
1






active

oldest

votes









active

oldest

votes






active

oldest

votes









0












$begingroup$

(1) is the definition.



Since the ordinals are well ordered, one just needs to show there is an uncountable ordinal.

Use Hartogs lemma. Alternatively, use the axiom of choice to well order the reals.



Let u be the first uncountable ordinal.

If k is a countable ordinal, k < u.

So u is an upper bound of countable ordinals.

If d is an upper bound of the countable ordinals and d < u, then d is countable. As d + 1 is countable, d + 1 <= d, a contradiction.

So u is the supremum of the countable ordinals.






share|cite|improve this answer









$endgroup$













  • $begingroup$
    We don't need AC . Let $B$ be the set of subsets of $omega times omega$ that are (graphs of) well-orderings. Each $bin B$ is isomorphic to a $unique$ (countable) ordinal $F(b).$ Then $ {F(b): bin B} =omega_1$.
    $endgroup$
    – DanielWainfleet
    Jan 26 at 16:58
















0












$begingroup$

(1) is the definition.



Since the ordinals are well ordered, one just needs to show there is an uncountable ordinal.

Use Hartogs lemma. Alternatively, use the axiom of choice to well order the reals.



Let u be the first uncountable ordinal.

If k is a countable ordinal, k < u.

So u is an upper bound of countable ordinals.

If d is an upper bound of the countable ordinals and d < u, then d is countable. As d + 1 is countable, d + 1 <= d, a contradiction.

So u is the supremum of the countable ordinals.






share|cite|improve this answer









$endgroup$













  • $begingroup$
    We don't need AC . Let $B$ be the set of subsets of $omega times omega$ that are (graphs of) well-orderings. Each $bin B$ is isomorphic to a $unique$ (countable) ordinal $F(b).$ Then $ {F(b): bin B} =omega_1$.
    $endgroup$
    – DanielWainfleet
    Jan 26 at 16:58














0












0








0





$begingroup$

(1) is the definition.



Since the ordinals are well ordered, one just needs to show there is an uncountable ordinal.

Use Hartogs lemma. Alternatively, use the axiom of choice to well order the reals.



Let u be the first uncountable ordinal.

If k is a countable ordinal, k < u.

So u is an upper bound of countable ordinals.

If d is an upper bound of the countable ordinals and d < u, then d is countable. As d + 1 is countable, d + 1 <= d, a contradiction.

So u is the supremum of the countable ordinals.






share|cite|improve this answer









$endgroup$



(1) is the definition.



Since the ordinals are well ordered, one just needs to show there is an uncountable ordinal.

Use Hartogs lemma. Alternatively, use the axiom of choice to well order the reals.



Let u be the first uncountable ordinal.

If k is a countable ordinal, k < u.

So u is an upper bound of countable ordinals.

If d is an upper bound of the countable ordinals and d < u, then d is countable. As d + 1 is countable, d + 1 <= d, a contradiction.

So u is the supremum of the countable ordinals.







share|cite|improve this answer












share|cite|improve this answer



share|cite|improve this answer










answered Jan 24 at 22:49









William ElliotWilliam Elliot

8,5922720




8,5922720












  • $begingroup$
    We don't need AC . Let $B$ be the set of subsets of $omega times omega$ that are (graphs of) well-orderings. Each $bin B$ is isomorphic to a $unique$ (countable) ordinal $F(b).$ Then $ {F(b): bin B} =omega_1$.
    $endgroup$
    – DanielWainfleet
    Jan 26 at 16:58


















  • $begingroup$
    We don't need AC . Let $B$ be the set of subsets of $omega times omega$ that are (graphs of) well-orderings. Each $bin B$ is isomorphic to a $unique$ (countable) ordinal $F(b).$ Then $ {F(b): bin B} =omega_1$.
    $endgroup$
    – DanielWainfleet
    Jan 26 at 16:58
















$begingroup$
We don't need AC . Let $B$ be the set of subsets of $omega times omega$ that are (graphs of) well-orderings. Each $bin B$ is isomorphic to a $unique$ (countable) ordinal $F(b).$ Then $ {F(b): bin B} =omega_1$.
$endgroup$
– DanielWainfleet
Jan 26 at 16:58




$begingroup$
We don't need AC . Let $B$ be the set of subsets of $omega times omega$ that are (graphs of) well-orderings. Each $bin B$ is isomorphic to a $unique$ (countable) ordinal $F(b).$ Then $ {F(b): bin B} =omega_1$.
$endgroup$
– DanielWainfleet
Jan 26 at 16:58


















draft saved

draft discarded




















































Thanks for contributing an answer to Mathematics Stack Exchange!


  • Please be sure to answer the question. Provide details and share your research!

But avoid



  • Asking for help, clarification, or responding to other answers.

  • Making statements based on opinion; back them up with references or personal experience.


Use MathJax to format equations. MathJax reference.


To learn more, see our tips on writing great answers.




draft saved


draft discarded














StackExchange.ready(
function () {
StackExchange.openid.initPostLogin('.new-post-login', 'https%3a%2f%2fmath.stackexchange.com%2fquestions%2f3086397%2fproperties-defintions-of-the-first-uncountable-ordinal%23new-answer', 'question_page');
}
);

Post as a guest















Required, but never shown





















































Required, but never shown














Required, but never shown












Required, but never shown







Required, but never shown

































Required, but never shown














Required, but never shown












Required, but never shown







Required, but never shown







Popular posts from this blog

Mario Kart Wii

What does “Dominus providebit” mean?

Antonio Litta Visconti Arese